Math, asked by aswanthika, 27 days ago

prove that :
tan (theta) / 1-cot (theta) +cot (theta) / 1-tan (theta) = 1+ sec (theta) cosec (theta)

Answers

Answered by PreetiGupta2006
21

 \large \bf \: Prove \:  that \:  :

 \sf \dfrac{ \tan \theta }{1 -  \cot\theta }  \:  -  \: \dfrac{  \cot  \theta }{1 -  \cot\theta }  \:  =  \: 1 \:  +  \:  \sec\theta  \:  cosec\theta

\: \large\: \bf \: So \:  Let's  \: begin \:  :

 \sf \: Solving \:  LHS

 \large\: \;\tt{ \;\;{L.H.S.\;=\;\bigg[\dfrac{\tan  \theta }{(1\:-\:\cot  \theta)}\bigg]\;+\;\bigg[\dfrac{\cot  \theta}{(1\:-\:\tan  \theta)}\bigg]}}

\large\: \;\tt{\longrightarrow\dfrac{\dfrac{\sin  \theta}{\cos  \theta}}{ 1 \:-\:\dfrac{\cos  \theta}{\sin  \theta}}\;+\;\dfrac{\dfrac{\cos  \theta}{\sin  \theta}}{ 1 \:-\:\dfrac{\sin  \theta}{\cos  \theta}}}

\large\: \;\tt {\longrightarrow\dfrac{\dfrac{\sin  \theta}{\cos  \theta}}{ \dfrac{1}{1} \:-\:\dfrac{\cos  \theta}{\sin  \theta}}\;+\;\dfrac{\dfrac{\cos  \theta}{\sin  \theta}}{ \dfrac{1}{1} \:-\:\dfrac{\sin  \theta}{\cos  \theta}}}

\large\: \;\tt{\longrightarrow\dfrac{\dfrac{\sin  \theta}{\cos  \theta}}{  \:\dfrac{ \sin \theta  \:  -  \: \cos  \theta}{\sin  \theta}}\;+\;\dfrac{\dfrac{\cos  \theta}{\sin  \theta}}{  \:\dfrac{ \cos \theta   \:  -  \: \sin  \theta}{\cos  \theta}}}

 \large\:  \sf \dfrac{\sin^{2} \theta}{cos\theta( \sin\theta -  \cos\theta) } \:  \:  +  \:  \dfrac{\ \cos^{2} \theta}{ \sin \theta( \ \cos \theta -  \ \sin \theta) }

\large\:  \sf \: \: Taking \:  -  \:  as \:  common.

\large\: \sf \dfrac{\sin^{2} \theta}{cos\theta( \sin\theta -  \cos\theta) } \:  \:  +  \:  \dfrac{\ \cos^{2} \theta}{  - \sin \theta( \ \ \sin\theta -  \ \ \cos \theta) }

\large\: \sf \dfrac{\sin^{2} \theta}{cos\theta( \sin\theta -  \cos\theta) } \:  \:   -  \:  \dfrac{\ \cos^{2} \theta}{  \sin \theta( \ \ \sin\theta -  \ \ \cos \theta) }

\large\:  \dfrac{1}{ \sin \theta \: -  \cos\theta }   \: \bigg[ \:  \dfrac{ \sin ^{2}  \theta}{ \cos \theta}  \:  -  \:  \dfrac{\cos^{2} \theta}{\sin   \theta}  \bigg]

 \large\: \sf \: Taking \:  LCM,  \: we  \: get

 \large\: \dfrac{1}{ \sin \theta \: -  \cos\theta } \:  \bigg[ \:  \dfrac{ \sin ^{3}  \theta  - \:  \cos ^{3}  \theta}{sin \theta  \:  cos \theta}  \bigg]

 \large\: \longrightarrow \sf \:  \: Formula \:  used :

 \large\: \boxed  {  \sf  \: a ^{3}  \:  -  \:b ^{3}  \:  =  \: (a \:  -  \: b)  \: (a^{2}  \:  + b ^{2} \:  +  \: ab )}

\large\: \dfrac{1}{ \sin \theta \: -  \cos\theta } \: \dfrac{(\sin\theta  - \cos\theta)(sin ^{2} +  \cos^{2} +  \sin\theta \: cos\theta) }{sin\theta\cos\theta}

\large\: \dfrac{1}{  \cancel{\sin \theta \: -  \cos\theta }} \: \dfrac{( \cancel{\sin\theta  - \cos\theta})(sin ^{2} +  \cos^{2} +  \sin\theta \: cos\theta) }{sin\theta\cos\theta}

\large\: \dfrac{(sin ^{2} +  \cos^{2} +  \sin\theta \: cos\theta) }{sin\theta\cos\theta}

\large\: \sf{ We \: know \: that (sin ^{2} +  \cos^{2} +  \sin\theta \: cos\theta) =1}

\sf \:  \large\: \dfrac{1 \:  +  \sin \theta \: \cos\theta }{\sin \theta \: \cos\theta}

\sf \: \large\: \dfrac{1}{ \sin \theta \: \cos \theta  }  \:  +  \: \dfrac{ \sin\theta \cos\theta  }{ \sin \theta \: \cos \theta  }

\sf \: \large \:\dfrac{1}{ \sin \theta \: \cos \theta  }  \:  +  \: \dfrac{ \cancel{ \sin\theta \cos\theta  }}{\cancel{ \sin \theta \: \cos \theta } }

\sf \: \large\: \dfrac{1}{ \sin \theta }  \:   \times \dfrac{1}{  \cos  \theta }  \:  +  \: 1

\sf \:\large \: LHS =  \sec \theta \:   \: \cos \theta+1

\sf \: \large \:RHS  = \: 1 \:  +  \:  \sec \theta \:   \: \cos \theta

\sf LHS \:  = \:RHS

\sf\:Hence\: proved

Attachments:
Similar questions